Reduce the rational expression to lowest terms. If it is already in lowest terms, enter the expression in the answer box. Also, specify any restrictions on the variable.y³ - 2y² - 9y + 18/y² + y - 6Rational expression in lowest terms:Variable restrictions for the original expression: y

Reduce The Rational Expression To Lowest Terms. If It Is Already In Lowest Terms, Enter The Expression

Answers

Answer 1

ANSWER

[tex]\begin{gathered} \text{ Rational expression in lowest terms: }y-3 \\ \\ \text{ Variable restrictions for the original expression: }y\ne2,-3 \end{gathered}[/tex]

EXPLANATION

We want to reduce the rational expression to the lowest terms:

[tex]\frac{y^3-2y^2-9y+18}{y^2+y-6}[/tex]

First, let us factor the denominator of the expression:

[tex]\begin{gathered} y^2+y-6 \\ \\ y^2+3y-2y-6 \\ \\ y(y+3)-2(y+3) \\ \\ (y-2)(y+3) \end{gathered}[/tex]

Now, we can test if the factors in the denominator are also the factors in the numerator.

To do this for (y - 2), substitute y = 2 in the numerator. If it is equal to 0, then, it is a factor:

[tex]\begin{gathered} (2)^3-2(2)^2-9(2)+18 \\ \\ 8-8-18+18 \\ \\ 0 \end{gathered}[/tex]

Since it is equal to 0, (y - 2) is a factor. Now, let us divide the numerator by (y -2):

We have simplified the numerator and now, we can factorize by the difference of two squares:

[tex]\begin{gathered} y^2-9 \\ \\ y^2-3^2 \\ \\ (y-3)(y+3) \end{gathered}[/tex]

Therefore, the simplified expression is:

[tex]\frac{(y-2)(y-3)(y+3)}{(y-2)(y+3)}[/tex]

Simplify further by dividing common terms. The expression becomes:

[tex]y-3[/tex]

That is the rational expression in the lowest terms.

To find the variable restrictions, set the denominator of the original expression to 0 and solve for y:

[tex]\begin{gathered} y^2+y-6=0 \\ \\ y^2+3y-2y-6=0 \\ \\ y(y+3)-2(y+3)=0 \\ \\ (y-2)(y+3)=0 \\ \\ y=2,\text{ }y=-3 \end{gathered}[/tex]

Those are the variable restrictions for the original expression.

Reduce The Rational Expression To Lowest Terms. If It Is Already In Lowest Terms, Enter The Expression

Related Questions

Simply the expression. (Cos x) (sec x)-(sin^2 x)

Answers

[tex]Cos^{2}x[/tex] is the solution for the expression [tex](Cosx)(secx) - sin^{2}x[/tex]

The given expression is:

[tex](Cosx)(secx) - sin^{2}x[/tex]

Recall form Pythagorean Identity that;

[tex]secx = \frac{1}{cosx}[/tex]

We apply this property to obtain;

[tex](cosx)(\frac{1}{cosx}) - sin^{2}x[/tex]

By simplifying we get that;

[tex]1 - sin^{2}x[/tex]

Recall from the Pythagorean, we identity that;

[tex]1 - sin^{2}x = cos^{2}x[/tex]

Hence the answer is  [tex]cos^{2}x[/tex]  is the solution for the expression [tex](Cosx)(secx) - sin^{2}x[/tex]

To learn more about expressions click here https://brainly.com/question/4344214

#SPJ9

Kobe is 2.07 metres tall.
Marcus is 1.79 metres tall.
Stephen is taller than Marcus by half the difference between Kobe's height and
Marcus's height.
How tall, in metres, is Stephen?

Answers

The height of Stephen is = 1.79 + 0.14 = 1.93 meters

What is unitary method ?

In its simplest form, the unitary procedure is used to determine the value of a single unit from a given multiple. How to calculate the value of one pen, for example, if 40 cost Rs. 400. To finish it, using the unitary method. Additionally, after the value of a single unit has been established, we can multiply that value by the quantity of additional units required to establish the value of the additional units. This is typically how the concepts of ratio and proportion are used.

Calculation

Step: 1

The height of Kobe is 2.07 meters

The height of Marcus is 1.79 meters

The difference between Kobe's height and Marcus's height = 2.07 - 1.79 meters

And its half is = 1/2 * 0.28 = 0.14 meters

Stephen is taller than Marcus by half the difference between Kobe's height and Marcus's height

The height of Stephen is = 1.79 + 0.14 = 1.93 meters

learn more about unitary method here :

brainly.com/question/22056199

#SPJ13

A football team played A games last year. They lost B of those games. What was their winning percentage

Answers

Answer:

(A - B)/A x 100%

Explanation:

If they played A games last year and lost B of those games, they win the rest of the games, so they win A - B games.

Then, the percentage is calculated as the number of games that they win over the total number of games multiplied by 100, so the winning percentage is

(A - B)/A x 100%

Use the power-reducing formulas to rewrite the expression in terms of first powers of the cosines of multiple angles.tan4(4x)

Answers

Solution

[tex]\begin{gathered} \tan^2(4x)=\frac{1-\cos(8x)}{1+\cos(8x)} \\ \\ \Rightarrow\tan^4(4x)=\frac{1-2\cos(8x)+\cos^2(8x)}{1+2\cos(8x)+\cos^2(8x)} \\ \\ \text{ since }\cos^2(8x)=\frac{1+\cos(16x)}{2} \\ \\ \Rightarrow\tan^4(4x)=\frac{1-2\cos(8x)+\frac{1+\cos(16x)}{2}}{1+2\cos(8x)+\frac{1+\cos(16x)}{2}} \\ \\ \Rightarrow\tan^4(4x)=\frac{2-4\cos(8x)+1+\cos(16x)}{2+4\cos(8x)+1+\cos(16x)} \\ \\ \Rightarrow\tan^4(4x)=\frac{3-4\cos(8x)+\cos(16x)}{3+4\cos(8x)+\cos(16x)} \end{gathered}[/tex]

The answer is:

[tex]\frac{3-4\cos(8x)+\cos(16x)}{3+4\cos(8x)+\cos(16x)}[/tex]

PLEASE HELP I’M IN MIDDLE SCHOOL AND I CANT FIND THE ANSWERS. ITS ALSO DUE IN A FEW HOURS.

Answers

To form a triangle the three angles must follow the inequality principle that says: The sum of any two sides must always be greater than the length of the third side. With this in mind let's check the angles.

For the first item we have:

[tex]\begin{gathered} 70+90>20\rightarrow160>20 \\ 70+20>90\rightarrow90>90 \\ 20+90>70\rightarrow110>90 \end{gathered}[/tex]

Since the second inequality is invalid the angles don't form a triangle.

For the second item we have:

[tex]\begin{gathered} 55+45>75\rightarrow100>75 \\ 55+75>45\rightarrow130>45 \\ 45+75>55\rightarrow120>55 \end{gathered}[/tex]

Since all the inequations are valid then the angles form a triangle. Since all the angles are smaller than 90 degrees, then this is an acute triangle.

For the third item we have:

[tex]\begin{gathered} 27+27>126\rightarrow54>126 \\ 27+126>27\rightarrow153>27 \end{gathered}[/tex]

Since the second inequation is invalid, then the angles don't form a triangle.

For the fourth item we have:

[tex]\begin{gathered} 38+87>55\rightarrow125>55 \\ 38+55>87\rightarrow93>87 \\ 55+87>38\rightarrow142>38 \end{gathered}[/tex]

Since all the inequalities are valid, then the angles form a triangle. All of its angles are smaller than 90 degrees, therefore this triangle is an acute triangle.

equation allows the nurse to use theratio and proportion method todetermine how many tablets the patientrequires?O 100 mg/50 mg x 1 tabletO 50 mg/100 mg x 1 tablet50 mg/1 tablet = 100 mg/ x tablets

Answers

SOLUTION

From the question, the patient needs 100 mg but the drug is available in 50 mg tablets.

We can see that 100 is twice of 50, so using 50 mg tablets should be twice that to make it 100 mg

If the number of tablets is given as x, so the requirement will be calculated as

[tex]x\text{ tablets = }\frac{1\text{ tablet }}{50\text{ mg}}\times100\text{ mg/1 tablet }[/tex]

Hence the last option is the correct answer

Use the table of values for f and g below to find the indicated compositions.f(g(8))=Answerg(f(5))=Answerf(f(4))=Answerg(g(2))=Answer

Answers

Starting from the first question, we want:

[tex]f(g(8))[/tex]

Let's start from the inside part:

[tex]g(8)[/tex]

Since the inside of "g" is 8, this means we need to check for the row with x = 8, thay is, "8" in the column "x". Finding this row, we check the value in this row and column "g", which is "4".

This means that:

[tex]g(8)=4[/tex]

Now, we can substitute this back in:

[tex]f(g(8))=f(4)[/tex]

And we can do similarly. We check the row which has "4" in the "x" column and see the corresponding value in column "f". We can see that it is also "4", so, the answer for the first is:

[tex]f(g(8))=f(4)=4[/tex]

Lastly, for the second, we do the same thing, the inside part is:

[tex]f(5)[/tex]

So, we got to line x = 5 and check column "f". It is 0, so:

[tex]\begin{gathered} f(5)=0 \\ g(f(5))=g(0)_{} \end{gathered}[/tex]

Now, we check row x = 0 and column "g" to find "9", so:

[tex]g(0)=9[/tex]

Thus, the answer for the second is:

[tex]g(f(5))=g(0)=9[/tex]

Transformation (x + 2, y - 3) is applied to triangle ABC.What are the coordinates of B’ (the transformation of point B)?A) (-5, 3)B) (2, - 1)C) (0, - 2)D) (2, - 3)

Answers

Transformation:

[tex](x+2,y-3)[/tex]

The coordinate of B in (x, y) we can get from the graph, which is (-2, 1):

Thus, the transformation based on the information given is:

[tex](-2+2,1-3)[/tex]

Simplifying:

[tex](0,-2)[/tex]

Answer: C) (0, -2)

A rectangular yard is 22 ft by 19 ft. The yard is covered with grass except for a square flower garden 10.5 ft long. How much grass is in the yard? The area covered by grass is (Type a whole number or a decimal.)

Answers

A rectangular yard is 22 ft by 19 ft. The yard is covered with grass except for a square flower garden 10.5 ft long. How much grass is in the yard? The area covered by grass is (Type a whole number or a decimal.) ​

we have that

The area covered by grass is equal to the area of a rectangular yard minus the area of the square flower garden

so

A=(22)(19)-(10.5)^2

A=307.75 ft2

Solve the equation by factoring. Separate multiple answers with a comma.

Answers

Answer:

x = -4, 0, 6

Explanations:

The given equation is:

[tex]x^3-24x=2x^2[/tex]

The equation can be re-written as:

[tex]x^3-2x^2\text{ - 24x = 0}[/tex][tex]\begin{gathered} x(x^2\text{ - 2x - 24) = 0} \\ x\lbrack x^2\text{ - 6x + 4x - 24\rbrack = 0} \\ x\lbrack x(x-6)\text{ + 4(x - 6)\rbrack = 0} \\ x\text{ (x - 6) (x + 4) = 0} \\ x\text{ = 0} \\ x\text{ - 6 = 0} \\ x\text{ = 6} \\ x\text{ + 4 = 0} \\ x\text{ = -4} \end{gathered}[/tex]

x = -4, 0, 6

Find the slope and y-intercept of the line.y = -3,000 + 30x

Answers

The given equation is

[tex]y=-3,000+30x[/tex]

It's important to know that this is a linear equation, and it's written in the form

[tex]y=mx+b[/tex]

Where m is the slope, and b is the y-intercept.

That means we only need to look for these values and that's it!

According to the given equation, we have

[tex]m=30,b=-3,000[/tex]Therefore, the slope is 30, and the y-intercept is at (0, -3000).

Show that when -9p2 + 4p + 1970 = 0, Total Revenue is at its maximum.Find the price and quantity which maximise Total Revenue.

Answers

Step 1

Write the demand function equation

[tex]Q=-9p^2+4p\text{ + 1970}[/tex]

Step 2:

To find the price and quantity which maximize the revenue

You will find the derivative of Q with respect to price

[tex]\begin{gathered} \frac{dQ}{dp}\text{ = -18p + 4} \\ -18p\text{ + 4 = 0} \\ 18p\text{ = 4} \\ p\text{ = }\frac{4}{18}\text{ = }\frac{2}{9} \end{gathered}[/tex]

Step 3:

Find the quantity demand by substituting p = 2/9

[tex]\begin{gathered} Q\text{ = -9 }\times\text{ (}\frac{2}{9})^2\text{ + 4 }\times\text{ }\frac{2}{9}\text{ + 1970} \\ =\text{ -0.44 + 0.888 + 1970} \\ =\text{ 1970.444} \\ =\text{ 1970} \end{gathered}[/tex]

Final answer

The price which maximizes the total revenue is p = 2/9

The quantity is Q = 1970

A cone-shaped pile of sawdust has a base diameter of 30 feet, and is 10 feet tall. Find the volume of the pile. Round your answer to the nearest tenth if necessary.

Answers

The formula for calculating the volume of a cone (pile) is expressed as:

[tex]V=\frac{1}{3}\pi r^2h[/tex]

r is the radius

h is the height of the cone (pile)

Given

Base diameter = 30 feet

radius = diameter/2

radius = 30/2

radius = 15 feet

height = 10feet

Required

Volume of the pile

To get the volume, you will substitute the given data into the formula as shown;

[tex]\begin{gathered} V\text{ = }\frac{1}{3}\pi(15)^2(10) \\ V\text{ = }\frac{1}{3}\pi(225)(10) \\ V=\frac{1}{3}(3.14)(2250) \end{gathered}[/tex]

Evaluate the final expression;

[tex]\begin{gathered} V\text{ =}\frac{7065}{3} \\ V\text{ = }2355ft^3 \end{gathered}[/tex]

Hence the volume of the pile is 2355ft^3

repeat the following procedure for the four given numbers. multiply the number by 2. add 10 to the product. divide the sum by 2. subtract 5 from the quotient. the first number is 2, the 2nd number is 4, the 3rd number is 8,the 4th number is 10

Answers

Lets start with the first case (the number is 2). Then, we get

Now, for the second case (the number is 4), the result is

Now, for 3rd case ( the number is 8), we have

And for 4th case (the number is 10) we get

a) As we can note, the result is always the same number we started with.

Then, if we represent the first number by n, the result is n.

b) Let n be the given number, then, we can write:

-Multiply the number by 2:

[tex]2n[/tex]

-Add 10 to the product:

[tex]2n+10[/tex]

- Divide the sum by 2:

[tex]\frac{2n+10}{2}[/tex]

but this is equal to

[tex]\frac{2n+10}{2}=\frac{2n}{2}+\frac{10}{2}=n+5[/tex]

- Substract 5 from the quotient:

[tex]\frac{2n+10}{2}-5[/tex]

but (from our last result) this is equal to

[tex]n+5-5=n[/tex]

which is our first number n. Thats why the answer in all cases is the same number we started with.

Kaira's gross pay is $6,820. Her deductions total $917.27. What percent of her grosspay is take-home pay?Round to the nearest whole percent.

Answers

Answer:

87%

Explanation:

• Kaira's gross pay = $6,820

,

• Deductions = $917.27

First, determine her take-home pay.

[tex]\begin{gathered} \text{Take}-\text{home Pay}=\text{Gross Pay-Deductions} \\ =6,820-917.27 \\ =\$5902.73 \end{gathered}[/tex]

Next, determine what percent of her gross pay is her take-home pay.

[tex]\begin{gathered} \text{Percent}=\frac{\text{Take}-\text{Home Pay}}{\text{Gross Pay}}\times100 \\ =\frac{5902.73}{6820}\times100 \\ =86.55\% \\ \approx87\% \end{gathered}[/tex]

87% of her gross pay is her take-home pay.

2. A shop owner raises the price of a $150 pair of shoes by 40%. After a few weeks,because of falling sales, the owner reduces the price of the shoes by 40%. Acustomer then says that the shoes are back at the original price.d. By what percent should the shoes be decreased in order to have the priceback at $150?Kk

Answers

Answer:

Explanations:

Given the following parameters;

Original price of shoe = $150

If the price is increased by 40%, the new price will be expressed as:

[tex]\begin{gathered} New\text{ price}=\$150+(0.40\times\$150) \\ New\text{ price}=\$150+\$60 \\ New\text{ price}=\$210 \end{gathered}[/tex]

If the the owner reduces the price of the shoes by 40% due to falling price, hence;

[tex]\begin{gathered} New\text{ price}=210-(0.4\times210) \\ New\text{ price}=210-84 \\ New\text{ price}=\$126 \end{gathered}[/tex]

Find the GCF of 24m^4n and 16m^2n.

Answers

Answer:

[tex]8m^2n[/tex]

Step-by-step explanation:

To find the GCF (greatest common factor), we have to find the prime factors of each number. Then, we have to find similar factors.

In this exercise, we have:

24m⁴n = 2 * 2 * 2 * 3 * m * m * m * m * n

16m²n = 2 * 2 * 2 * 2 * m * m * n

The GCF will be 2 * 2 * 2 * m * m * n

So, The GCF is 8m²n.

create a linear equation in the slope-intercept form that contains points (2,8) and (6,-4)

Answers

the two points are,

A(2,8)

B (6,-4)

the equation of the line in the slope intercept form is,

[tex]y-8=\frac{-4-8}{6-2}(x-2)[/tex][tex]\begin{gathered} y-8=\frac{-12}{4}(x-2) \\ y-8=-3x+6 \\ y=-3x+14 \end{gathered}[/tex]

thus, the equation of the line is

y = -3x + 14

Drag each label to the correct location on the image. Are these functions even, odd , or neither?

Answers

We know that if a function is even then the graph of the function is symmetrical about y-axis.

Look at the first function, it is symmetrical about y-axis, henve it is even,

if a function is odd then the graph of the function is symmetrical about origin

Look at the second function, it is symmetrical about origin, henve it is odd,

And the third function is neither symmetrical about y-axis nor symmetrical about origin and so it neither odd nor even.

Answer:

see photo

Step-by-step explanation:

Plato/Edmentum

what is 12 times 12?

Answers

Answer:

144

Step-by-step explanation:

Answer: 12x12=144.

I hope this helps!!!!!!!!!!!!!!!!!!!!!!!!!!

Only need help with finding the mean and standard deviation.

Answers

Given

Probability distribution table

Find

Mean and standard deviation

Explanation

mean for probability distribution is given by

[tex]mean=\sum_^xP(x)[/tex]

so , mean

[tex]\begin{gathered} 0+0.25+0.54+0.36+0.64 \\ 1.79 \end{gathered}[/tex]

standaed deviation

[tex]\begin{gathered} \sum_^x^2P(x) \\ 0+0.25+1.08+1.08+2.56 \\ 4.97 \end{gathered}[/tex]

Final Answer

mean = 1.79

standard deviation = 4.97

I really need help. volume of this figure using 3.14 without rounding.

Answers

We get that the volume of the figure is the volume for the rectangular prism and the cylinder with radius 4

[tex]\begin{gathered} V=16\cdot11\cdot11+3.14\cdot4^2\cdot9 \\ =1936+452.16 \\ =2388.16 \end{gathered}[/tex]

Determine the signs of given trigonometric function of an angle in standard position with given measure.cos (-302°) and sin (-302°)

Answers

Explanation

Exercise: Calculate the sign of cos(-302°) and sin(-302°).

We can know the sign of the trigonometric value of an angle depending on which quadrant the angle is located. In the case of cosine and sine, we have the following rules:

Let's calculate the sign of cos(-302°) first. Note that the angle within the cosine is negative; this means that it must be measured from the x-axis clockwise. Now, every quadrant has a total measure of 90°; then, dividing 302 by 90 we obtain

[tex]\frac{302}{90}\approx3.4,[/tex]

which means that -302° goes through two complete quadrants and a partial part of a third one. Since we must measure the angle clockwise, -302 lies in the first quadrant.

By the diagram above, the sign of -302° is +.

On the other hand, looking at the sine diagram, we see that the sign of sine(-302°) is + as well.

Answer

The signs of cos(-302°) and sin(-302°) are both +.

The table below represents the data collected at a sandwich shop for the last six months withrespect to the type of bread people ordered (sourdough or wheat) and whether or not theygot cheese on their sandwich.With cheeseWithout cheeseSourdough800425Wheat1200700What is the P(cheese | wheat)? Show all work to receive full credit. You can place your workin this box or attach it in the box on the final question.

Answers

Given:

The table represents the data collected at a sandwich shop for the last six months with respect to the type of bread

We will find P(cheese | wheat)

We will use the following formula:

[tex]P(A|B)=\frac{P(A\cap B)}{P(B)}[/tex]

Let A represents cheese, and B represents wheat

From the table:

[tex]\begin{gathered} A\cap B=1200 \\ B=1200+700=1900 \end{gathered}[/tex]

So, the probability will be as follows;

[tex]P\left(cheese|wheat\right)=\frac{1200}{1900}=\frac{12}{19}[/tex]

So, the answer will be:

P(cheese | wheat) = 12/19 = 0.63

Circle O has a center at (2,-2) and a diameter of 8 units. Identify which point lies on Circle O. O A. (0,1) O B. (6,-2) O C. (2, 2) O D. (-3,-2)

Answers

In this case, we'll have to carry out several steps to find the solution.

Step 01:

Data

circle

center (2,-2)

diameter = 8

point on the circle = ?

Step 02:

equation of a circle

(x - a)² + (y - b)² = r²

r = d / 2 = 8 / 2 = 4

(x - 2)² + ( y - (-2))² = 4²

(x - 2)² + ( y + 2)² = 16

Step 03:

We must verify the points with the equation.

A. If (0 , 1)

(x - 2)² + ( y + 2)² = 16

( 0 - 2)² + ( 1 + 2)² = 16

4 + 9 = 16

15 < 16 the point is inside of the circle

B. If ( 6, -2)

(x - 2)² + ( y + 2)² = 16

(6 - 2)² + ( -2 + 2)² = 16

16 = 16 the point is on the circle

The answer is:

B. (6 , -2)

Solve this quadratic equation and explains the steps to help you solve it . Also which two specific methods help you to solve it: Is it quadratic formula, Factoring, Square Root Property or Completing square. And explains.

Answers

Recall that the quadratic formula states that, the solutions to the quadratic equation

[tex]ax^2+bx+c=0[/tex]

are

[tex]\frac{-b\pm\sqrt{(-b)^2-4ac}}{2a}.[/tex]

Now, in the given equation, a=-1 ,b=-6, and c= 8. Therefore, the solutions to the equation are:

[tex]x=\frac{6\pm\sqrt{36+32}}{-2}.[/tex]

Simplifying the above result, we get:

[tex]x=-3\pm(-1)\sqrt{17}.[/tex]

Answer:

[tex]\begin{gathered} x_1=-3+\sqrt{17,} \\ x_2=-3-\sqrt{17}. \end{gathered}[/tex]

Describe the transformations: F(x) = - 3(x +1)^2 + 5 What is the Parent Function: Opens UP or DOWN:Stretch or Shrink: Vertical Movement Up or Down, how many units:Horizontal Movement Left or Right, how many units:

Answers

You have the following transformation:

[tex]F(x)=-3(x+1)^2+5[/tex]

The previous function if a parabola. the Parent Function of any parabola is:

[tex]y=x^2[/tex]

All others parabolas can be obtained by applying transformation to the Parent Function.

If you expand the parenthesis of F(x), you obtain:

[tex]\begin{gathered} F(x)=-3(x^2+2x+1)+5=-3x^2-6x-3+5 \\ F(x)=-3x^2-6x+2 \end{gathered}[/tex]

The dominant term, that is, the term with the variable x powered to 2, has a negative coefficient, it demands that the parabola open DOWN.

The coefficient of the dominant term is -3, then I-3I = 3 > 1. It means that the function stretches away from the x-axis.

The function streches way from x-axis by a factor of 3 units.

Vertical movement Up with 5 units. This is becasue the constant of the function, 5, which means the vertical translation of the Pater Function, if the constant is positive the translation is upward, if the translation is negative, it is downward.

Horizontal translation Left with 1 untit. The horizontal trasnlation is given by the constant term inside the quadratic parenthesis, in this case is + 1, whichi represents a translation of the graph to the left.

hey! I need help/answers to 20 questions. Heres the first question Find the volume of a pyramid with a square base, where the side length of the base is 10.9 m and the height of the pyramid is 4.7 m. Round your answer to the nearest tenth of a cubic meter.

Answers

Answer:

The Volume of the pyramid is;

[tex]186.1\text{ }m^3[/tex]

Explanation:

Given the pyramid with a square base and dimensions;

[tex]\begin{gathered} l=10.9m \\ h=\text{4}.7m \end{gathered}[/tex]

Recall that the volume of a pyramid can be calculated using the formula;

[tex]V=\frac{l\times l\times h}{3}[/tex]

Substituting the given values;

[tex]\begin{gathered} V=\frac{10.9\times10.9\times4.7}{3} \\ V=186.1m^3 \end{gathered}[/tex]

Therefore, the Volume of the pyramid is;

[tex]186.1\text{ }m^3[/tex]

an airplane is flying at an altitude of 5000 ft in starseed sitting at a rate of 150 ft per minute what we'd altitude of the plane after 10 minutes

Answers

Problem

An airplane is flying at an altitude of 5000 ft in starseed sitting at a rate of 150 ft per minute what we'd altitude of the plane after 10 minutes

Solution

For this case we can define the following notation:

y= altitude

x= time in minutes

We can find the altitude at any time with the following formula:

y = 5000 - 150 x

And we can find the value for the altitude at x=10 we got:

y = 5000 -150(10) =3500 ft

is a projectile is launched from a height of 5 m with an intial velocity of 25 meters per second, how many seconds will it take the projectile to hit the ground

Answers

In this problem we know that the initial velocity in the horizontal axis is 25, however the initial velocity in the y axis will be o so we can use this equation:

[tex]y=y_0+v_it+\frac{1}{2}at^2[/tex]

So to find the time the object get the grownd we can replace the initial high for 5 miter, the final one for 0 and the aceleration by -9.8 (gravity) so:

[tex]0=5+0(t)+\frac{1}{2}(-9.8)t^2[/tex]

and we solve for t so:

[tex]\begin{gathered} 9.8t^2=5\cdot2 \\ t^2=\frac{10}{9.8} \\ t\approx\sqrt[]{1} \\ t\approx1 \end{gathered}[/tex]

So the projectil takes one secon to hit the grownd

Other Questions
Which can be the first step in finding the equation of the line passing through (5,-4) and (-1,8) use two unit multipliers to convert 500 feet to centimeters Solve for x.4.5(x+3.21)=33.21Responses x = 4.17 x, = 4.17 x = 6.67 x, = 6.67 x = 8.09 x, = 8.09 x = 10.59 A new car worth $27,000 is depreciating in value by $3,000 per year. After how many years will the car's value be $3,000? #14 iW Ua. Give another name for UVb. Name a ray with endpoint Xc. Match each ray with its opposite ray. b. Calculate the electric force that exists between two objects that are 0.500 m apart and carrycharges of 0.00450 C and 0.00240 If 3.4 atm of a gas at 644.4 K is compressed to 11.6 atm at constant volume. What is the new pressure of the gas? Write an equation in point slope form for the line that passes through (-3,5) with a slope of -3 A guidebook says that one night at a mid-range hotel the capital city, republica cost between $25 an $40 us dollars. The hotel capital city offers a one week rental for 150 rp ( republica pounds) the current exchange rate is 1=0.0147 usd ($us dollars) does the price per night at the hotel capitol suggest that it a mid range hotel? The diagram shows a tile in the shape of a trapezium.12 cm10 cmWork out the area of the tile.26 cm solve the equation by completing the square.4x(x + 6) = -40 of the exterior angle of a regular octagon measures (11x+1), fond the value of x. Consider the following graph of two functions.(8-1-2)Step 3 of 4: Find (8.5(-2)Enable Zoom/Pan866) = -3010-35SG) = x + 3 g in glycolysis, mutase catalyzes a more or less energetically neutral reaction that results in the transfer of a functional group from one position to another. Last year at a certain high school, there were 140 boys on the honor roll and 68 girls on the honor roll. This year, the number of boys on the honor roll increased by 5% and the number of girls on the honor roll increased by 25%. By what percentage did the total number of students on the honor roll increase? Round your answer to the nearest tenth (if necessary). which theory would support the idea that by increasing the efficiency of resource utilization in the global economy, international trade results in greater economic growth and provides economic benefits to all countries that participate in a global trading system? need help asap will give you 5 stars! need a quick worker! Write the equation of the line in slope-intercept form. Through the points (-8, 15) and (6, 15) Which binomial is a factor of 49x2 + 84xy + 36y2?. 7 + 6. x+6yC. - 6D. 7- 6PLEASE HELP ASAP I WILL GIVE YOU BRAINLIEST!!!!!! triangle QRS is reflected across the y axis to create triangle QRS complete the rule that describes the coordinates of triangle QRS after the reflection has occurred